Solving Integral Problems: Step-by-Step Guide for Definite Integrals in Calculus

  • Thread starter Thread starter Learner123
  • Start date Start date
Click For Summary
The discussion revolves around solving a definite integral problem involving the bounds defined by y = e^x, y = √(1 - x^2), and x = 1. The user initially misused the closed line integral symbol instead of the standard integral sign. They encountered difficulties with the integration by parts method, particularly with the term involving xe^(2x). Clarification was provided that the entire expression must be evaluated at the endpoints for definite integrals. The user expressed gratitude for the assistance in understanding this key concept.
Learner123
Messages
16
Reaction score
0
Intergral problem! please help!

Homework Statement


\oint(x: 0 to 1)\oint(y: \sqrt{}(1 - x^2) to e\overline{}x) xydydx

The region bounded by y = e\overline{}x, y = \sqrt{}(1 - x^2), and x =1
3. The Attempt at a Solution
i got stuck when i came to the part: 1/2 \oint(x: 0 to 1) (e^(2x) -1 + x^2)xdx
i appreciate any help
 
Physics news on Phys.org


The symbol you are using is the symbol for a closed line integral. You should be using a normal integral sign: \int.
Otherwise, since the integral is a linear operator, you have the following sum of integrals:
\frac{1}{2}\left(\int xe^{2x} dx - \int x dx + \int x^3 dx\right)
Which one is giving you a problem?
 


the first one xe^(2x) thing
i guess it's intergral by part, but not sure
 


I tried to do part and this is how i done (for the first intergral):
u = x, du = dx, v = 1/2e^(2x), dv = e^(2x)dx
uv - \int vdu
1/2xe^(2x) - \int 1/2e^(2x)dx
1/2xe^(2x) - 1/4(e^2 -1 )
x runs from 0 to 1, but 1/2xe^(2x) is not in the intergral part, so how to eliminate x?
very appreciate for more help!
 


Learner123 said:
1/2xe^(2x) - 1/4(e^2 -1 )
This entire expression is the indefinite integral; the entire expression must be evaluated at the endpoints of the integral if the integral is definite.
 


got it! i didn't know that after spending 3 calculus classes, what a shame of me! thank you so much for your help and your time.
 
Question: A clock's minute hand has length 4 and its hour hand has length 3. What is the distance between the tips at the moment when it is increasing most rapidly?(Putnam Exam Question) Answer: Making assumption that both the hands moves at constant angular velocities, the answer is ## \sqrt{7} .## But don't you think this assumption is somewhat doubtful and wrong?

Similar threads

  • · Replies 14 ·
Replies
14
Views
2K
  • · Replies 105 ·
4
Replies
105
Views
6K
  • · Replies 10 ·
Replies
10
Views
2K
Replies
4
Views
2K
  • · Replies 10 ·
Replies
10
Views
2K
  • · Replies 3 ·
Replies
3
Views
2K
  • · Replies 12 ·
Replies
12
Views
2K
  • · Replies 2 ·
Replies
2
Views
2K
  • · Replies 2 ·
Replies
2
Views
2K
  • · Replies 4 ·
Replies
4
Views
2K